ارسال ها
337
لایک ها
82
امتیاز
0
#21
پاسخ : ماراتن نظریه اعداد (سطح مقدماتی)

پاسخت کامل نیست
 

darya.f

New Member
ارسال ها
182
لایک ها
114
امتیاز
0
#22
پاسخ : ماراتن نظریه اعداد (سطح مقدماتی)

مىشه ىه جواب دىگه بگىن به طور مثال ببىنم چه فرمىه؟
 
ارسال ها
337
لایک ها
82
امتیاز
0
#23
پاسخ : ماراتن نظریه اعداد (سطح مقدماتی)


البته شما جوابتون کامل بنویسید که چطور به این دو دسته جواب رسیدید ماراتن باید یه کم کامل تر باشه فک کنم:116:
 
آخرین ویرایش توسط مدیر

math1998

New Member
ارسال ها
336
لایک ها
224
امتیاز
0
#24
پاسخ : ماراتن نظریه اعداد (سطح مقدماتی)

سوال بعد : (لطفا شما هم سوال بذارید)
معادله ی زیر را حل کنید . (یه کم سطحش بالا تر از سوال های بالاست اما فعلا سعیم اینه که تعداد زیادی سوال اسون و سخت و متوسط مطرح بشه به ترتیب سختی )






دیگه راحت شد راحت دست میاد .
 
ارسال ها
337
لایک ها
82
امتیاز
0
#25
پاسخ : ماراتن نظریه اعداد (سطح مقدماتی)

راه حل شما مشکل داره چون اگر
از طرفی از عبارت پایین هم نیجه میده که x=2d+1 که یک جواب ناقص است به نظر من غلط های دیگه ای هم داره اما این فقط نظر منه بازم خودت بررسی کن
 

darya.f

New Member
ارسال ها
182
لایک ها
114
امتیاز
0
#26
پاسخ : ماراتن نظریه اعداد (سطح مقدماتی)

راه حل شما مشکل داره چون اگر
از طرفی از عبارت پایین هم نیجه میده که x=2d+1 که یک جواب ناقص است به نظر من غلط های دیگه ای هم داره اما این فقط نظر منه بازم خودت بررسی کن
ببخشىد منظورتون راه حل math1998 ه؟
 

AHZolfaghari

Well-Known Member
ارسال ها
935
لایک ها
1,654
امتیاز
93
#27
پاسخ : ماراتن نظریه اعداد (سطح مقدماتی)

سوال بعد : (لطفا شما هم سوال بذارید)
معادله ی زیر را حل کنید . (یه کم سطحش بالا تر از سوال های بالاست اما فعلا سعیم اینه که تعداد زیادی سوال اسون و سخت و متوسط مطرح بشه به ترتیب سختی )




دو عدد x,y نمی تونن به پیمانه 4 هم نهشت باشن پس فرض میکنیم که یکی شون به پیمانه 4 باشه سه و دیگری باشه یک .
چون x+y توانی از دو هستش پس نسبت به اعداد فرد اول هست


فرض کنید x به پیمانه 4 بشه یک پس x+1 فقط یه عامل دو داره به عبارت ساده تر میشه 2 ضرب در یه عبارت فرد پس بنابر لم اقلیدس اون عبارت فرد رو میشه حذف کرد پس داریم:

حالا اگه x+y برابر با 2x-2 نباشه پس حداقل نصفش هست یعنی
که آشکارا تناقض دارد پس برابر با خودش هست یعنی :

پس

که اگه به فرض سوال نگاه کنید صدق می کنه

این سوال بعد :

همه اعداد اول p و اعداد صحیح x,y را بیابید بطوریکه :



 

math1998

New Member
ارسال ها
336
لایک ها
224
امتیاز
0
#28
پاسخ : ماراتن نظریه اعداد (سطح مقدماتی)

راه حل شما مشکل داره چون اگر
از طرفی از عبارت پایین هم نیجه میده که x=2d+1 که یک جواب ناقص است به نظر من غلط های دیگه ای هم داره اما این فقط نظر منه بازم خودت بررسی کن
ببخشید توان xy+1 برابر P .
 

Dadgarnia

New Member
ارسال ها
1,350
لایک ها
1,127
امتیاز
0
#29
پاسخ : ماراتن نظریه اعداد (سطح مقدماتی)

این سوال بعد :

همه اعداد اول p و اعداد صحیح x,y را بیابید بطوریکه :



واضح است كه
و p عددي فرد است. با كم كردن دو رابطه از هم داريم:

از طرف ديگر مي دانيم:
پس بنابر لم اقليدس داريم:

حالا به راحتي بدست مي آيد
 
ارسال ها
337
لایک ها
82
امتیاز
0
#30
پاسخ : ماراتن نظریه اعداد (سطح مقدماتی)

ببخشىد منظورتون راه حل math1998 ه؟
بله راه حل ایشون به نظر من مشکل داره

---- دو نوشته به هم متصل شده است ----





دو عدد x,y نمی تونن به پیمانه 4 هم نهشت باشن پس فرض میکنیم که یکی شون به پیمانه 4 باشه سه و دیگری باشه یک .
چون x+y توانی از دو هستش پس نسبت به اعداد فرد اول هست


فرض کنید x به پیمانه 4 بشه یک پس x+1 فقط یه عامل دو داره به عبارت ساده تر میشه 2 ضرب در یه عبارت فرد پس بنابر لم اقلیدس اون عبارت فرد رو میشه حذف کرد پس داریم:

حالا اگه x+y برابر با 2x-2 نباشه پس حداقل نصفش هست یعنی
که آشکارا تناقض دارد پس برابر با خودش هست یعنی :

پس

که اگه به فرض سوال نگاه کنید صدق می کنه

این سوال بعد :

همه اعداد اول p و اعداد صحیح x,y را بیابید بطوریکه :



چرا x,y نمیتونن به پیمانه چهار هم نهشت باشن ؟ مگه 1,1 جواب مساله نیست. بازم ناقصه اما از نظر من قابل قبول. حالا اگه کسی ایراد داشت بگه

---- دو نوشته به هم متصل شده است ----

سوال بعد. معادله زیر را حل کنید. دوستان هر کی سوال حل کرد یه سوال بذاره وجدانا.

x,y طبیعی اند
 

math1998

New Member
ارسال ها
336
لایک ها
224
امتیاز
0
#31
پاسخ : ماراتن نظریه اعداد (سطح مقدماتی)

بله راه حل ایشون به نظر من مشکل داره

---- دو نوشته به هم متصل شده است ----


چرا x,y نمیتونن به پیمانه چهار هم نهشت باشن ؟ مگه 1,1 جواب مساله نیست. بازم ناقصه اما از نظر من قابل قبول. حالا اگه کسی ایراد داشت بگه

---- دو نوشته به هم متصل شده است ----

سوال بعد. معادله زیر را حل کنید. دوستان هر کی سوال حل کرد یه سوال بذاره وجدانا.

x,y طبیعی اند
2001=3.23.29
ابتدا ثابت میکنیم x عاملی جز 3و23و29 ندارد حال فرض کنیم عاملی به جز این 3 عدد مانند p[SUB]k[/SUB] دارد.


اما میدانیم همچین چیزی امکان ندارد پس a[SUB]k[/SUB]=0 پس تناقض است پس x و y هیچ عاملی به جز 3و23و29 ندارد.





حال به راحتی نتیجه میشه x=y پس x=y=2001
حالت دوم نیز راحت بدست میاد زیرا چون x مقسوم علیه 2001 است اگر بجای x مقسوم علیه های 2001 را قرار دهیم یک جواب برای y بدست میاد پس در کل 8 جواب دارد .
 

Dadgarnia

New Member
ارسال ها
1,350
لایک ها
1,127
امتیاز
0
#32
پاسخ : ماراتن نظریه اعداد (سطح مقدماتی)

2001=3.23.29
ابتدا ثابت میکنیم x عاملی جز 3و23و29 ندارد حال فرض کنیم عاملی به جز این 3 عدد مانند p[SUB]k[/SUB] دارد.


اما میدانیم همچین چیزی امکان ندارد پس a[SUB]k[/SUB]=0 پس تناقض است پس x و y هیچ عاملی به جز 3و23و29 ندارد.





حال به راحتی نتیجه میشه x=y پس x=y=2001
حالت دوم نیز راحت بدست میاد زیرا چون x مقسوم علیه 2001 است اگر بجای x مقسوم علیه های 2001 را قرار دهیم یک جواب برای y بدست میاد پس در کل 8 جواب دارد .
چرا
؟ فكر كنم اين درست باشه:
 

math1998

New Member
ارسال ها
336
لایک ها
224
امتیاز
0
#33
پاسخ : ماراتن نظریه اعداد (سطح مقدماتی)

a[SUB]k[/SUB]2001=b[SUB]k[/SUB]p[SUB]k[/SUB][SUP]a[SUB]k[/SUB][/SUP]

---- دو نوشته به هم متصل شده است ----

چرا
؟ فكر كنم اين درست باشه:
اره خوب اما p[SUB]k[/SUB][SUP]a[SUB]k[/SUB][/SUP] هم x رو عاد میکنه پس a[SUB]k[/SUB]2001 رو هم عاد میکنه اشتباه من بود اصلاح شد.
 
آخرین ویرایش توسط مدیر

AHZolfaghari

Well-Known Member
ارسال ها
935
لایک ها
1,654
امتیاز
93
#34
پاسخ : ماراتن نظریه اعداد (سطح مقدماتی)

بله راه حل ایشون به نظر من مشکل داره

---- دو نوشته به هم متصل شده است ----


چرا x,y نمیتونن به پیمانه چهار هم نهشت باشن ؟ مگه 1,1 جواب مساله نیست. بازم ناقصه اما از نظر من قابل قبول. حالا اگه کسی ایراد داشت بگه

---- دو نوشته به هم متصل شده است ----

سوال بعد. معادله زیر را حل کنید. دوستان هر کی سوال حل کرد یه سوال بذاره وجدانا.

x,y طبیعی اند
اگه هم نشهت باشن که خب n فقط دو میتونه باشه . دیگه اینم نوشتن میخواد ؟؟!؟!؟!!!!!!!!!!!!!!!!!!
دیگه بدیهیات راه حل رو که نباید بنویسم ! درسته مرحله دو باید نوشت اما اینجا سایت هست و فرق داره
 

math1998

New Member
ارسال ها
336
لایک ها
224
امتیاز
0
#35
پاسخ : ماراتن نظریه اعداد (سطح مقدماتی)

سوال بعد. همه ی مجموعه های ناتهی و متناهی s را بیابید که اگر
انوقت
 
ارسال ها
337
لایک ها
82
امتیاز
0
#36
پاسخ : ماراتن نظریه اعداد (سطح مقدماتی)

2001=3.23.29
ابتدا ثابت میکنیم x عاملی جز 3و23و29 ندارد حال فرض کنیم عاملی به جز این 3 عدد مانند p[SUB]k[/SUB] دارد.


اما میدانیم همچین چیزی امکان ندارد پس a[SUB]k[/SUB]=0 پس تناقض است پس x و y هیچ عاملی به جز 3و23و29 ندارد.





حال به راحتی نتیجه میشه x=y پس x=y=2001
حالت دوم نیز راحت بدست میاد زیرا چون x مقسوم علیه 2001 است اگر بجای x مقسوم علیه های 2001 را قرار دهیم یک جواب برای y بدست میاد پس در کل 8 جواب دارد .
چرا تو حالت اول 2001 x رو عاد میکنه؟ جواب درسته اما راه حل کمی ایراد داره به نظر من

---- دو نوشته به هم متصل شده است ----

سطح سوال زیاد بالانبر pls
راهنمایی ثابت کنید که دو حتما عضو مجموعه هست بقیشم بدیهیه (برای سوال اقای math1998)
سوال بعد بدم یا بیشتر صبر کنیم ؟
 

darya.f

New Member
ارسال ها
182
لایک ها
114
امتیاز
0
#37
پاسخ : ماراتن نظریه اعداد (سطح مقدماتی)

چرا تو حالت اول 2001 x رو عاد میکنه؟ جواب درسته اما راه حل کمی ایراد داره به نظر من

---- دو نوشته به هم متصل شده است ----

سطح سوال زیاد بالانبر pls
راهنمایی ثابت کنید که دو حتما عضو مجموعه هست بقیشم بدیهیه (برای سوال اقای math1998)
سوال بعد بدم یا بیشتر صبر کنیم ؟
اگه مىشه ىکم سر هر سوال بىشتر صبر کنىم...
 

math1998

New Member
ارسال ها
336
لایک ها
224
امتیاز
0
#38
پاسخ : ماراتن نظریه اعداد (سطح مقدماتی)

چرا تو حالت اول 2001 x رو عاد میکنه؟ جواب درسته اما راه حل کمی ایراد داره به نظر من

---- دو نوشته به هم متصل شده است ----

سطح سوال زیاد بالانبر pls
راهنمایی ثابت کنید که دو حتما عضو مجموعه هست بقیشم بدیهیه (برای سوال اقای math1998)
سوال بعد بدم یا بیشتر صبر کنیم ؟
اثبات کردم که x عاملی به جز 3و23و29 نداره تازه اگه فرض کنیم یکی از این عوامل رو نداشته باشه و بزرگتر از 2001 باشه جواب نداره که راحت میشه چک کرد دیگه تمام راه حل که نیاز نیست فقط کلیتش مهمه اگه بخوام همشو بنویسم میدونی چقدر زمان میبره مهم اینه که معادله 8 جواب داره که بدستشون هم اوردم
درضمن بی زحمت تاپیکای هندسه و جبر و ترکیبیات هم راه بنداید
 
آخرین ویرایش توسط مدیر

darya.f

New Member
ارسال ها
182
لایک ها
114
امتیاز
0
#39
پاسخ : ماراتن نظریه اعداد (سطح مقدماتی)

چرا تو حالت اول 2001 x رو عاد میکنه؟ جواب درسته اما راه حل کمی ایراد داره به نظر من

---- دو نوشته به هم متصل شده است ----

سطح سوال زیاد بالانبر pls
راهنمایی ثابت کنید که دو حتما عضو مجموعه هست بقیشم بدیهیه (برای سوال اقای math1998)
سوال بعد بدم یا بیشتر صبر کنیم ؟
سوال بعد رو بزارىن دىگه!
 
ارسال ها
337
لایک ها
82
امتیاز
0
#40
پاسخ : ماراتن نظریه اعداد (سطح مقدماتی)

جواب های معادله زیر را بیابید

x,y طبیعی اند
 
بالا